- PowerScore Staff
- Posts: 5972
- Joined: Mar 25, 2011
- Sun Jun 01, 2014 11:50 am
#87492
Complete Question Explanation
(The complete setup for this game can be found here: lsat/viewtopic.php?f=150&t=2097)
The correct answer choice is (B)
The question stem requires you to identify a pair of variables where at least one of the two members must always volunteer. The two most obvious answers are the pairs created by the third and fourth rules—S and V, and R and L—because during the setup we identified that each was a pair where at least one member volunteered. However, none of the answers identified contains either of the two pairs. Thus, we must look further into this problem to find the correct answer.
In the third and fourth rules the circumstance that created the result requested in this question was a negative sufficient condition paired with a positive necessary condition. In the setup discussion regarding the long chain inference, we mentioned that a number of sub-inferences could be drawn regarding the relationship of L and other variables. One such relationship is that of L and M:
If L does not volunteer, then M must volunteer, and via the contrapositive if M does not volunteer then L must volunteer. Thus, at least one of L and M must always volunteer. Accordingly, answer choice (B) is correct.
Note that answer choice (C) is incorrect because the relationship between L and V is as follows:
Thus, neither volunteers or both volunteer.
(The complete setup for this game can be found here: lsat/viewtopic.php?f=150&t=2097)
The correct answer choice is (B)
The question stem requires you to identify a pair of variables where at least one of the two members must always volunteer. The two most obvious answers are the pairs created by the third and fourth rules—S and V, and R and L—because during the setup we identified that each was a pair where at least one member volunteered. However, none of the answers identified contains either of the two pairs. Thus, we must look further into this problem to find the correct answer.
In the third and fourth rules the circumstance that created the result requested in this question was a negative sufficient condition paired with a positive necessary condition. In the setup discussion regarding the long chain inference, we mentioned that a number of sub-inferences could be drawn regarding the relationship of L and other variables. One such relationship is that of L and M:
If L does not volunteer, then M must volunteer, and via the contrapositive if M does not volunteer then L must volunteer. Thus, at least one of L and M must always volunteer. Accordingly, answer choice (B) is correct.
Note that answer choice (C) is incorrect because the relationship between L and V is as follows:
Thus, neither volunteers or both volunteer.
You do not have the required permissions to view the files attached to this post.
Dave Killoran
PowerScore Test Preparation
Follow me on X/Twitter at http://twitter.com/DaveKilloran
My LSAT Articles: http://blog.powerscore.com/lsat/author/dave-killoran
PowerScore Podcast: http://www.powerscore.com/lsat/podcast/
PowerScore Test Preparation
Follow me on X/Twitter at http://twitter.com/DaveKilloran
My LSAT Articles: http://blog.powerscore.com/lsat/author/dave-killoran
PowerScore Podcast: http://www.powerscore.com/lsat/podcast/